Laplace Transform

That sure is a nice problem you've got there. But what are your thoughts? What have you tried? Please re-read the Read Before Posting thread that's stickied at the top of each sub-forum and comply with the rules found therein. In particular, please share with us any and all work you've done on this problem, even the parts you know for sure are wrong. Thank you.
 
Top